The smart way to learn. The smart way to teach.

MRCOG PART 2 SBAs and EMQs

Course PAID
notes336
EMQ1502
SBA2115
Do you realy want to delete this discussion?
Forum >>

Essay 357

Posted by A 4.
The clinical assessment will start with obtaining a history from the patient. Details on menstrual regularity and last menstrual period will help identify the gestational age. Contraceptive history is equally important as ectopic pregnancies are more common with IUDs.      Details about her current relationship (e.g. length and stability), history of of STIs and PID, or previous pregnancies (including ectopics) will help in risk assessment. Surgical history in the abdomen or pelvis, or any abdominal inflammatory conditions (IBS, IBD, Endometriosis etc) are further risk factors. On examination, initially, I should check that the patient is stable, responsive, and that her vital signs are normal (no tachycardia or hypotension) Abdominally I’ll check for guarding, tenderness, rebound tenderness or signs or Puritanism. Pelvic examination should start with a speculum assessment to check the situation of the cervix (opened or bleeding) and obtain triple swabs. Bimanually, I shall assess the uterine size and mobility, any adnexal fulness and any cervical excitation. Finally, part of my assessment should include introducing an intravenous line and obtaining a full blood count (FBC), group and save (G&S)and HCG blood samples. B. The types of investigations depend in the clinical severity. A urine test can exclude a urinary tract infection. Basic investigations in the firm of FBC, G&S and HCG are essential as we need to obtain an idea on any drop in the haemoglobin or evidence of infection (leucocytosis). HCG will offer an objective assessment of the pregnancy. If there is a suspicion of infection, CRP and ESR should be included, while if there is suspicion of intraabdominal bleeding then crossmatching of 4 units of RBCs is essential. If there is suspicion that she is deteriorating, then renal and liver functions need to be assessed. A clotting profile will help if she is unwell. If the patient is stable, then we can arrange for a TA/TV ultrasound and await for the FBC and HCG. However, if she starts getting worse then a diagnostic laparoscopy will be the next step. C. There are several points to be taken in consideration before starting medical treatment. Firstly, the patient should be stable with no symptoms. Moreover, the HCG levels should be less than 3000 units. If the pregnancy was identified by ultrasound, then it should not exceed the 30mm in size. The patient, additionally, should understand that is is a long term treatment, it may need repeating and it may fail with the need of surgical treatment. Patient’s compliance is important as she will need to have repeated blood tests frequently and must avoid getting pregnant before the condition is resolved. Finally, the unit should have the appropriate facilities to deal with any forthcoming situations 24 hours a day.
Posted by m T.

(A) I would ask her about her parity and her history of previous miscarriages and ectopic pregnancies. I would ask her if this is a planned pregnancy and if any assisted conception method was used because it is associated with increased chance of ectopic pregnancy. I would ask her about her history of sexually transmitted disease and past history of surgery to assess her risk of ectopic pregnancy. I would ask her if she is using any contraception like intrauterine contraceptive device would increased her chance of ectopic pregnancy and if she was using any barrier methods like the condom to prevent sexually transmitted infection. I would ask her about any other associated symptoms like fever, vomiting, diarrhoea, dysuria to look for other causes of abdominal pain. I would ask her about the history of cervical smear as cervical pathology can present with bleeding in pregnancy.

I would examine her abdomen looking for signs of acute abdomen. A speculum examination is indicated to look at the cervix, to check if the os is opened and to look at the amount of bleeding.

I also need to know her blood pressure and pulse to assess her stability.

 

(B) In order to quantify the amount of blood loss and to assess if she needs blood transfusion, a full blood count looking at the haemoglobin would be useful. Platelet count and coagulation studies would be useful to check for evidence of disseminated intravascular coagulopathy. Blood group is needed because if she is rhesus negative, she would need anti-D immunoglobulin if she undergoes any form of medical or surgical management for miscarriage or ectopic pregnancy. A group and cross match of blood is required in the event that transfusion would be needed as she is actively bleeding. A renal panel and liver panal should be done as basic investigation. An ultrasound is needed to assess the viability of the pregnancy and to determine the location of the pregnancy. It can also assess if there is evidence of intraperitoneal bleeding by looking at the amount of fluid in the pouch of Douglas. A transvaginal ultrasound would be indicated if a transabdominal scan is unable to visualize the pregnancy. If her cervix looks suspicious, a PAP smear or biopsy of the cervix would be required. She needs to be screened for Chlamydia infection because she is 23 years old. If there is suspicion of other causes of abdominal pain like appendicitis, further investigations need to be considered like an abdominal x-ray or CT scan.

 

 

(C) Expectant management should only be considered if the patient has minimal or no symptoms and is clinically stable. If the patient has only mild abdominal pain, without requiring analgesia, her bHCG is less than 1000 iu/l, if there is less than 100ml of blood in the pouch of Douglas and there is no fetal cardiac activity seen, expectant management can be considered. However, if she describes the pain as severe or she has heavy vaginal bleeding or is clinically unstable, expectant management will not be an option. The advantage of expectant management would be avoiding a surgery and the need to undergo general anaesthesia. The disadvantage is that she will need close follow up and active interventions will be needed if she becomes symptomatic or her serum bHCG rises above 1000 iu/l. If the patient is clinically unwell, she should be offered surgical management instead of expectant management. A diagnostic laparoscopy should be done and a unilateral salpingectomy done if there is evidence of ectopic pregnancy in the tube and the other tube is normal. If the patient is unstable, a laparotomy might be a faster method.

Posted by Emma S.

A healthy 23 year old woman attends the early pregnancy clinic with a 6 hour history of lower abdominal pain and vaginal bleeding. Her LMP was 8 weeks ago and she had a positive pregnancy test 2 weeks ago. Her pregnancy test is positive on admission. (a) Discuss your clinical assessment [7 marks]. (b) Discuss the factors that will influence your investigations [7 marks]. (c) She is thought to have an ectopic pregnancy and is offered expectant management. Can this approach be justified? [6 marks] 

The diagnosis here is an ectopic pregnancy until proven otherwise. It is important to establish that she is clinically stable before taking a complete history so a full set of observations to allow the early warning score to be calcuated should be performed (blood pressure, pulse rate, temperature, respiratory rate and oxygen saturations). Once we have established that she is stable a history should be taken to narrow down the differential diagnoses such as ectopic pregnancy, miscarriage, ovarian cyst accident, appendicitis, UTI and pelvic inflammatory disease (PID).

A general medical and surgical history is needed as they may impact on future treatment as she may have had a salpingectomy for a previous ectopic pregnancy. Risk factors for an ectopic pregnancy such as previous ectopic pregnancy or PID should be determined. A past history of any other pregnacies and their outcomes and plans for future pregnancies may also impact on treatment decisions. We need to establish if this is a wanted pregnancy and whether she is at risk of STIs if so referral to the GUM clinic is appropriate if accepted.

An examination should include abdominal palpation looking for rebound or guarding to suggest peritonism or any pelvic masses. A bimanual examination may reveal an adnexal mass or tenderness. A speculum examination to assess the cervix and to obtain triple swabs (with consent) should be performed.

 

Investigations will depend on how haemodynamically stable the patient is and what is acceptable for her. She should have a blood test taken for beta-hcg, group and save, full blood count and progesterone . If she is unstable she should be taken immediately to theatre. Otherwise, If she is stable and has a beta-hcg level below the discriminatory level according to the local hospital guideline (usually 1000-1500) she could be managed as an out-patient following extensive counselling, to return in 48 hours for a repeat serum beta-hcg. If she lives far from the hospital or lives alone then admission would be safer. If the beta-hcg is above the discriminatory level or there signs suggestive of other pathology then an ultrasound scan, usually transvaginal should be performed looking for any evidence of an ectopic or intrauterine pregnancy. If clinical examination points to other pathology the referral to another specialist is appropriate.

 

Ectopic pregnancy can be managed conservatively so long as the patient fits the strict criteria including minimal pain and bleeding, no free fluid on scan and no fetal cariac activity. The patient must be willing to attend regularly for follw up until the beta hcg is less than 20. In this case conservative management can only be justified after clarifying her symptoms, scan findings and beta hcg level..

The patient should be given the options of medical managment with methotrexate, providing that the beta hcg is less than 3000, there is no free fluid, adnexal mass is less than 4cm, she is willing to attend for follow up and there are no medical contraindications to the drug suuch as liver disease. She should be fully counselled and receive written information.

She should also be given the option of surgical management. The procedure should be explained, including the risks and benefits and implications for future pregnancies. The laparoscopic approach should be used as lomg as the surgeon has the appropriate skills and equipment. The contralateral tube should be assessed and if normal a salpingectomy should be performed. If the contralateral tube is abnormal then salpingotomy can be attempted and the patient followed up with beta hcgs.

Essay 357 Posted by Deepa S.

A healthy 23 year old woman attends the early pregnancy clinic with a 6 hour history of lower abdominal pain and vaginal bleeding. Her LMP was 8 weeks ago and she had a positive pregnancy test 2 weeks ago. Her pregnancy test is positive on admission.

(a) Discuss your clinical assessment [7 marks].

Any woman of reproductive age presenting with a positive pregnancy test, abdominal pain and vaginal bleeding is suspected for ectopic pregnancy unless proven otherwise.  Ectopic pregnancy is a life threatening condition but preventable when diagnosed early and with appropriate treatment. In the given clinical scenario my assessment will involve a brief history with abdomen and pelvic examination. In history , risk factors such as tubal factors involving previous ectopic pregnancy , tubal surgery, pelvic infection specifically Chlamydia as it remains the most common cause for ectopic pregnancy must be excluded. Contraception history such as IUCD insertion, as itself is associated with salpingitis causing secondary tubal damage. Progesterone only pill is also thought to predispose ectopic pregnancy. History of stable relationship as otherwise associated risk for STI’s which can predispose to tubal pregnancy. Description of pain either unilateral or generalized pain, subdiaphragmatic irritation from intra-abdominal bleed results in shoulder tip pain/ collapse. The colour and amount of bleeding must be assessed as commonly minimal bleeding is noted in 75% of cases, occasionally passing of decidual cast is mistaken for products of conception. Vital signs including temperature, pulse, blood pressure, oxygen saturation must be recorded. Signs of tachycardia or hypotension suggest cardiovascular compromise warrant surgical treatment. Clinical signs of acute abdomen – peritonism from haemoperitoneum can cause guarding. Cervical excitation may be present when there is blood in pelvis and cervical os can be assessed to rule out miscarriage. Adnexal mass may be palpable if it is a large ectopic pregnancy

(b) Discuss the factors that will influence your investigations [7 marks].

Although most women are haemodynamically stable, the nature of the condition is unpredictable and they deteriorate quickly therefore large bore IV access for resuscitation if required and bloods should be sent at the time for FBC, Group and save – to check rhesus status/antibodies or crossmatch blood and call for senior support, alert anaesthetic and theatre staff if hemodynamic compromise. Serum beta HCG and progesterone can be helpful if scan is inconclusive. A progesterone of >60 mmol/l is suggestive of viable pregnancy conversely < 20 mmol/l suggests failing pregnancy. Serum beta HCG more than 1500 iu is the cut off level for ultrasound scan transvaginal or transabdominal . All inconclusive scan should be managed clinically primarily. In those deemed to be stable with no cardiovascular compromise and inconclusive scan a repeat beta HCG levels should be done in 48 hrs apart to monitor the rate of rise of HCG level. In normal pregnancy at least double every 48 hrs or over 66 % increase from previous HCG result.It may double or remain suboptimal in ectopic pregnancy.

(c) She is thought to have an ectopic pregnancy and is offered expectant management. Can this approach be justified? [6 marks] 

Expectant management is acceptable form of treatment for ectopic pregnancy in selected cases. Women should be clinically stable, asymptomatic, an ultrasound diagnosis of ectopic pregnancy with no evidence or less than 100 mls of blood in the pouch of Douglas and a decreasing serum hCG, initially less thanserum 1000 iu/l. It is however only acceptable if it involves minimal risks to the woman. Women selected for expectant management should be counselled about the importance of compliance with follow-up and should be within easy access to the hospital if any concerns. Women managed expectantly should be followed twice weekly with serial hCG measurements and weekly by transvaginal examinations to ensure a rapidly decreasing hCG level (ideally less than 50% of its initial level within seven days) and a reduction in the size of adnexal mass by seven days. Thereafter, weekly hCG and transvaginal ultrasound examinations are advised until serum hCG levels are less than 20 iu/l as there is possibility of tubal rupture even at low levels of βhCG.

Essay 357 Posted by Deepa S.

 a)  Any woman of reproductive age presenting with a positive pregnancy test, abdominal pain and vaginal bleeding is suspected for ectopic pregnancy unless proven otherwise.  Ectopic pregnancy is a life threatening condition but preventable when diagnosed early and with appropriate treatment. In the given clinical scenario my assessment will involve a brief history with abdomen and pelvic examination. In history , risk factors such as tubal factors involving previous ectopic pregnancy , tubal surgery, pelvic infection specifically Chlamydia as it remains the most common cause for ectopic pregnancy must be excluded. Contraception history such as IUCD insertion, as itself is associated with salpingitis causing secondary tubal damage. Progesterone only pill is also thought to predispose ectopic pregnancy. History of stable relationship as otherwise associated risk for STI’s which can predispose to tubal pregnancy. Description of pain either unilateral or generalized pain, subdiaphragmatic irritation from intra-abdominal bleed results in shoulder tip pain/ collapse. The colour and amount of bleeding must be assessed as commonly minimal bleeding is noted in 75% of cases, occasionally passing of decidual cast is mistaken for products of conception. Vital signs including temperature, pulse, blood pressure, oxygen saturation must be recorded. Signs of tachycardia or hypotension suggest cardiovascular compromise warrant surgical treatment. Clinical signs of acute abdomen – peritonism from haemoperitoneum can cause guarding. Cervical excitation may be present when there is blood in pelvis and cervical os can be assessed to rule out miscarriage. Adnexal mass may be palpable if it is a large ectopic pregnancy.

 b) Although most women are haemodynamically stable, the nature of the condition is unpredictable and they deteriorate quickly therefore large bore IV access for resuscitation if required and bloods should be sent at the time for FBC, Group and save – to check rhesus status/antibodies or crossmatch blood and call for senior support, alert anaesthetic and theatre staff if hemodynamic compromise. Serum beta HCG and progesterone can be helpful if scan is inconclusive. A progesterone of >60 mmol/l is suggestive of viable pregnancy conversely < 20 mmol/l suggests failing pregnancy. Serum beta HCG more than 1500 iu is the cut off level for ultrasound scan transvaginal or transabdominal . All inconclusive scan should be managed clinically primarily. In those deemed to be stable with no cardiovascular compromise and inconclusive scan a repeat beta HCG levels should be done in 48 hrs apart to monitor the rate of rise of HCG level. In normal pregnancy at least double every 48 hrs or over 66 % increase from previous HCG result.

 c) Expectant management is acceptable form of treatment for ectopic pregnancy in selected cases. Women should be clinically stable, asymptomatic, an ultrasound diagnosis of ectopic pregnancy with no evidence or less than 100 mls of blood in the pouch of Douglas and a decreasing serum hCG, initially less thanserum 1000 iu/l. It is however only acceptable if it involves minimal risks to the woman. Women selected for expectant management should be counselled about the importance of compliance with follow-up and should be within easy access to the hospital if any concerns. Women managed expectantly should be followed twice weekly with serial hCG measurements and weekly by transvaginal examinations to ensure a rapidly decreasing hCG level (ideally less than 50% of its initial level within seven days) and a reduction in the size of adnexal mass by seven days. Thereafter, weekly hCG and transvaginal ultrasound examinations are advised until serum hCG levels are less than 20 iu/l as there is possibility of tubal rupture even at low levels of βhCG.

Posted by kunal R.
History of severity duration of abdominal pain, fever vomiting, dizziness, fainting episodes, Passage of products mass blood clots, intermittent pain and contarction. History of sexually transmitted infections and treated for the same, current iucd user, past history of tubal surgery. General examination to assess hemodynamic stability of patient , pallor, tachycardia, tachypnic, temperature. Abdominal examination to assess severity of tenderness localized or generalized suggesting rupture of ectopic pregnancy , distension , presence of absence of bowel sounds. Vaginal examination for cervical motion tenderness, clots, products, adnexal mass. Chlamydia test offered as patients age less than 25 years. B) Investigations include levels of serum bhcg and it's correlation with gestational age, levels more than 3000 suggesting intrauterine pregnancy or ectopic pregnancy that favors medical or surgical treatment between 1500to 3000 favouring medical treatment of ectoic pregnancy or less than 1500 favoring expectant or medical treatment, rise of bhcg levels, Hemoglobin concentration , blood group and save,Transabdominal, transvaginal scan to note where the gestational sac is intrauterine or extrauterine, size of pregnancy sac less than 4 cm or more than that, free fluid in abdomen, ascites,presence of fetal heart beats. Rh status of patient as in negative pregnancy patient requires 250 iu of anti d to prevent sensitization. If the clinical condition suggest ectopic or ruptured ectopic pregnancy and diagnosis is in doubt diagnostic followed at the same time operative laparoscopic treatment after considering patient decision choice and consent. Hemodynamic unsuitability warrants expedite treatment. C) Patient explained the condition and written evidence based information given if possible with leaflet or diagrammatic view of extrauterine pregnancy. Patient told that there are three options for treatment of extrauterine pregnancy in tube.This includes expectant ( wait and monitor treatment) , medical treatment with injection methotrexate and surgical management either done laparoscopically or laparotomy. Treatment options depends on the general condition of the patient, hemo dynamic stability if present, levels of bhcg more than 1500, fetal heart beats present, blood in abdomen or ruptured cases does not favor expectant management. Expectant management done when Sr bhcg levels less than 1500, falling titres , size of sac less than 3-4 cm. Sr bhcg levels done initially every 48hr till the end of first week. Fall in titres should be by more than 15 % tha initial value on repetition and weekly trasvaginal scan in which the diameter of sac should decrease by 50% than previuos sac diameter. With expectant management there is less risk to patient of surgery, anesthetic risk , iatrogenic complication while surgery, less potential side effects of inj methotrexate.x 29% of patient fails to respond via expectant management and require further treatment via medical or surgical. 75%will complain of intermittent abdominal pain. Post treatment patient given contraceptive advice and To report to epau when pregnant next time as she misses her period by 6 weeks.
Posted by Muthu M.
  1. As the patient is at risk of ectopic pregnancy, threatened miscarriage, or inevitable miscarriage, so I would like to make sure she is clinically stable.  I will start with quick history of pain score and bleeding amount, dizziness and history of shoulder tip pain. The pain whether generalized or unilateral lower abdominal pain to point towards threatened vs. ectopic.   I would like to know, whether it is a planned pregnancy or using any contraception such as Progesterone only pill, which may be putting her at risk of ectopic pregnancy.  I would like to other risk factors for ectopic such as any previous history of Pelvic inflammatory disease and endometriosis. Also ask for previous obstetric history and ectopic pregnancy history.   I like to check her pulse and blood pressure, if there is tachycardia and low blood pressure, pale looking and tachypnoea means that patient is clinically compromised.  Also, check temperature, oxygen saturation to rule out other risks of inflammatory causes such as appendicitis, pelvic infections for her pain.  I would like to know when she last ate and drink and any other previous abdominal surgeries and infertility treatment to assess if surgery is needed immediately.  I like to examine the abdomen to check for localized area of tenderness, guarding and rebound tenderness, distention to rule out acute abdomen, internal bleeding.  I will do speculum examination to check amount of bleeding and the opening of cervical os or products at the os to rule out inevitable miscarriage vs. threatened miscarriage.  I will also do a bimanual examination, to check size of the uterus and adnexal masses and tenderness to check for the possibility of ectopic pregnancy.
  2. If patient is clinically stable, anyone who is referred to Early pregnancy assessment unit should have a transvaginal scan of pelvis to look for the location of pregnancy and viability or size / weeks of the pregnancy.  If no pregnancy is seen inside the uterus it should be supported with Serum BHCG level and may be progesterone levels and a follow-up plan.  The BHCG level will help in such a way that if it is very low or less than 1000IU, it may be too early to visualize the pregnancy.  Meanwhile if more than 1000IU, it raises the possibility of ectopic pregnancy or pregnancy of unknown location or complete miscarriage.  I will also do Full blood count to check the haemoglobulin status to help to decide clinical stability and blood group and save: for checking Rh status, as she may need Anti D if any intervention occurred and also to facilitate crossmatch if low Hb with clinically compromised situation or surgical intervention occurs.  If she is unwell and clinically compromised she would require either laparoscopy or laparotomy  or Examination under anesthesia / evacuation of retained products of conception depends on the situation.

3. It is justifiable approach, provided the patient is clinically stable, had been given with written information / consent regarding the diagnosis, follow-up plan and would understand there is still possibility of surgical intervention may be still needed, understands the risks of medical management with methotexrate.  The early pregnancy unit should have a treatment and followup protocol for doing medical management.  There should be dedicated team of gynaecologist, early pregnancy unit staff nurse and sonographer available and the patient would be able to contact them at any time of the day and have immediate access if any complication arises such as signs of rupture: sudden onset of pain and clinicall very unwell.  We should not do medical management, if by scan, there is large amount of free fluid in the pouch of douglas as it may have ruptured already, should not be viable ectopic as the success rate of treatment is not good.   We have to make sure that BHCG level is less than 3000iu, and the gestational sac size is smaller to have a good success rate with this treatment.    Patient must be reliable & compliant and will definitely attend the early pregnancy unit for follow-up as per protocol and would be contactable from the unit if necessary.

EARLY PREGNANCY COMPLICATIONS Posted by NADIRA RAHMAN N.

A healthy 23 year old woman attends the early pregnancy clinic with a 6 hour history of lower abdominal pain and vaginal bleeding. Her LMP was 8 weeks ago and she had a positive pregnancy test 2 weeks ago. Her pregnancy test is positive on admission. (a) Discuss your clinical assessment [7 marks]. (b) Discuss the factors that will influence your investigations [7 marks]. (c) She is thought to have an ectopic pregnancy and is offered expectant management. Can this approach be justified? [6 marks]

(a)Initial assessment:

Hx:

Duration and severity of symptoms:type of pain-colicky or spasmodic,severity,radiation,amount of bleeding.

Menstrual history:accuracy of LMP,regularity of cycle,duration,dysmenorrea shud b sout for.

Obstetric history:Para,gravida,previous pregnancy outcome,previous ectopic pregnancy,incomplete abortion,MR,hydatidiform mole.

Contraceptives:Barrier use,future fertility wishes.

 

Previous gynaecological history:sexually transmitted infections,PID,partner"s history,

 

Examination:

General-Pulse,Blood pressure,anaemic status,respiration

Abdominal-symmphysiofundal height may b disproportionately inceased in hydatidiform mole,hypogastric area tendernessin threatened or incomplete abortion,adnexal tenderness in ectopic pregnancy.

Pervaginal:Gentle perspeculum shud done to see dilatation of cervix or product hanging in incomplete abortion,slight bleeding in threatened or ectopic pregnancy.Cervical motion test positive in ectopic pregnancy.

 

Investigation:

Blood grouping &Rh typing,Hb%,cross match should b done.

TVS-Gestational age of fetus,viability,intrauterine or extrauterine pregnancy,molar gestation

Serum Beta HCG

Urine RME & culture sensitivity test.

 

(b)Factors that will influence investigations:

Anaemic status-Blood grouping &Rh typing,Hb%,Cross match bloodas per anaemic status.

Rh (-)ve blood group:Kleihauer test.

Disproportionate symphysiofundal height suggestive of mole ,Severe adnexal tenderness and previous history of STD or ectopic suggestive of ectopic pregnancy-Serum beta HCG

Early gestation with vaginal bleeding and lower abdominal pain:transvaginal sonography(TVS)-To differentiate between abortion,ectopic,mole.

S.beta HCG below discriminatory zone-Repeatation in 48 hours.More than 66%rise chance of early intrauterine pregnancy,shud repeat TVS 7days.Less than66% rise chance of ectopic gestation.

 (c) She is thought to have an ectopic pregnancy and is offered expectant management. Can this approach be justified?

If she has S.beta HCG below discriminatory zone(1000-1500IU/L) and value after48hours rises <66%chance of ectopic gestation.She can b a candidate for expectant Mx if her beta HCG level<1000IU/L,pouch of douglas fluid collection<100ml,hemodynamically stable,less sign-symptoms,small adnexal mass and no cardiac pulsation.She can b discharge at home provided she understands the value of repeated investigations like s.beta HCG 2to3 times/week  until value<100IU/L and weekly TVS to detect size of adnexal mass.Expectant Mx is also justified on this background that she agrees to admit and surgical Mx of ectopic pregnancy if S.beta HCG value rises or s/s aggravates.Written documentation of the counselling should b kept and she should have 24hours emergency contact numbersof the hospital.

 

Posted by C M.

A)

 My initial assessment would include taking a thorough history and clinical examination. I would ask about the nature of the pain (duration, constant, colicky, any radiation and alleviating or aggravating factor). Whether she experiences shoulder tip pain with dizziness, fainting or collapse is also important as it could help discern between possible internal haemorrhage and miscarriage. I would clarify her parity and confirm her LMP paying attention to whether her periods were regular or irregular. Her current method of contraception if she uses any is also important as the use of an IUD or minipill may make her at higher risk of an ectopic pregnancy. I would also enquire about any past abdominal surgeries (previous appendicitis – as this is also a differential diagnosis- or previous ectopic pregnancies as well as and PID or STD’s in the past). On clinical examination basic observations would be of importance i.e. BP. Pulse, Temp, Respiratory rate and Sats – because if she is haemodynamically unstable i.e. tachycardia, tachypnoeic with shallow breaths and hypotensive she would need to go to theatre straight away for a laparotomy.  I would also assess her general condition namely if she looks pale and conscious as this also has bearing on the possible diagnosis. An abdominal examination would help identify and acute abdomen from a non acute abdomen so assessing for rebound tenderness, guarding and rigidity as well as a distended abdomen. Identification of this warrants theatre whereas if she is having a miscarriage then this would not be the case. A vaginal examination is also warranted to see if there are products in the cervical os, to assess that amount of bleeding and to see if the cervical os was open. I would ensure she has IV access and admit her.

 

B)

Firstly if she was bleeding heavily I would arrange for a FBC and Groups and save because I would want to see her Hb and Plts as well as have a group and save ready to cross match if she warranted a blood transfusion. If however she was haemodynamically unstable I would automatically cross match 4 units of blood and consider transfusing straight away. If there is evidence of hemodynamic compromise then a U&E as well as LFT and Clotting is warranted as renal failure occur with sever haemorrhage as well as DIC hence the clotting and LFT. As she is younger than 25 I would take a vaginal swab for Chlamydia and Gonorrhoea on vaginal examination with her consent, this is considered good practice for young people. If on examination I identify an open os I would arrange for a transvagainal ultrasound to see if there are any products of conception in the uterus. If however the os is closed and there is minimal to moderate bleeding then an abdominal and transvaginal ultrasound would be indicate to rule out an ectopic pregnancy as well as identify if this is a miscarriage. A serum BHCG is indicated as this may help decide how we manage her – if we manage her expectantly then a baseline BHCG is warranted or if she has miscarried then the next sample would be considerably reduced.

 

C)

Expectant management of ectopic pregnancies is usually indicated if the patient is haemodynamically stable and asymptomptomatic as well as BHCG<1000 and falling. She must also be compliant and happy to come in regularly for BHCG checks and regular ultrasound scans. This lady is symptomatic with pain and bleeding and this would not be an ideal candidate for expectant management and she could easily deteriorate further. The only way that this approach may be justified is if she had very minimal pain and PV spotting and was adamant that she would not want to have methotrexate or ideally a laparoscopy then this would be the only course of action as she would need to consent to the other options.

 

Answer to Essay 357 Posted by Drxyz A.

 

Essay No. 357

A healthy 23 year old woman attends the early pregnancy clinic with a 6 hour history of lower abdominal pain and vaginal bleeding. Her LMP was 8 weeks ago and she had a positive pregnancy test 2 weeks ago. Her pregnancy test is positive on admission. (a) Discuss your clinical assessment [7 marks]. (b) Discuss the factors that will influence your investigations [7 marks]. (c) She is thought to have an ectopic pregnancy and is offered expectant management. Can this approach be justified? [6 marks]

 

Answer

(a)

Detailed history about the lower abdominal pain, type of pain and radiation to other organ, associate with nausea and vomiting and any fainting attack will be taken. She will be asked about vaginal bleeding, amount of bleeding, colour (old or fresh) and association with passage of clot. Urinary or bowel symptoms will be asked. Menstrual history will be taken regarding pattern, regularity and LMP. She will be asked about the  use of ovulation induction drugs. Previous obstetrics history will be inquired regarding ectopic pregnancy and miscarriage. She will be asked about the contraception. She will be asked about any history of vaginal infections and visit to GUM. Her general condition will be assessed for anemia. Her BP, Pulse and Temperature will be noted. Abdomen will be examined for any mass, tenderness, rebound tenderness and rigidity. Speculum examination to see amount of bleeding or POC. Vaginal examination to assess the cervical excitation and tenderness.

(b)

In case of suspicion of ruptured ectopic or miscarriage, general condition of the patient will give information about the need to do FBC. If there is low Hb, blood can be Group & save and cross matched. Rh Factor is important to decided Anti-D injection in Rh –ve patient. If ectopic pregnancy is suspected, transvaginal USG will be planned to see any adnexal mass or fluid in the cul-de sac in the absence of intra-uterine pregnancy. Serum beta HCG will be done to evaluate treatment option and follow up. Serum Progesterone with beta HCG  and TVS may help to diagnose ectopic  pregnancy. Endo-cervical vaginal swabs for chlamydia and gonorrhea will be taken in suspected acute PID. Laproscopy may be planned in some cases where the diagnosis is not certain.

c)

Expectant management in ectopic pregnancy is justified who are asymptomatic or minimal symptoms, their beta HCG < 1000 i.u/L and fluid in the cul-de sacn is < 100 ml. this approach is justified in the centers where 24 hour telephone and emergency services are available. These centers must have protocols to deal with these type of patients. Patient should be compliant to follow up. She must be fully aware for need of intervention and follow up. She will be called after one week and beta HCG should be decreased at least 50% and adnexal mass should be decreased in size. Beta HCG will be followed until < 20 iu/L. Patient information sheet will be given and informed consent for expectant management will be taken.

Management of early pregnancy bleeding Posted by Shradha G.

 

 
A healthy 23 year old woman attends the early pregnancy clinic with a 6 hour history of lower abdominal pain and vaginal bleeding. Her LMP was 8 weeks ago and she had a positive pregnancy test 2 weeks ago. Her pregnancy test is positive on admission. (a) Discuss your clinical assessment [7 marks]. (b) Discuss the factors that will influence your investigations [7 marks]. (c) She is thought to have an ectopic pregnancy and is offered expectant management. Can this approach be justified? [6 marks]
 
 
 
A) Clinical assessment of this patient includes thorough history and examination. History has to be taken regarding the eliciting cause of bleeding in early pregnancy i.e, trauma, fever, coitus, hyperemesis, syncope, passage of fleshy mass per vaginum and coagulopathy. History regarding regularity of periods and confirmation of gestation has to be enquired, contraceptive history, previous obstetric history in view of previous LSCS, curettage, ectopic pregnancy,molar pregnancy is also important to elicit. Past history of PID, tuberculosis, smoking and drug history also needs to be enquired. 
 
                    Thorough general, per abdomen and per vaginum examination is important to have a clue for the bleeding. Temperature, pulse ,BP, pallor, icterus, any petechiae, ecchymosis has to be reported. Per abdomen examination is of paramount importance; inspect the abdomen for any fullness in flanks, distension, palpate if abdomen is soft or guarding, rigidity is there, any tender point, if uterus is palpable or not.Sterile speculum examination has to be done to see the amount of bleeding, presence of any product of conception through os, status of os i.e, os closed or dilated, any foul smelling discharge. Gentle per vaginum examination is done to note the size of uterus if correspond to gestational age or not, os closed or dilated, any adnexal mass to rule out ectopic pregnancy.
 
 
B) Factors that will influence our investigations include the amount of bleeding, coexistent pallor; then FBC, blood group, crossmatch has to be done as  blood transfusion may be  required. Rise of temperature,presence of fever signifies CRP, urine routine microscopy during this period. High vaginal swab needs to be done to rule out the genital infection, if any leakage of liquor or foul discharge is there.Irregular periods,history of passage of fleshy mass, adnexal mass, previous LSCS, IUCD used as contraceptive warrants urgent TVS. TVS has better resolution than TAS. Look for the presence of intrauterine pregnancy, yolk sac, amnion, CRL, FHR. If these findingsare not there, look for blghted ovum, missed abortion, ectopic pregnancy, molar pregnancy. Colour doppler may provide an aid for the correct diagnosis,if ectopic or molar pregnancy is suspected on TVS. Serum Beta HCG carries important value in early pregnancy bleeding if fetal viability is in doubt during TVS or ectopic is suspected. Repeat BHCG after 48 hours further to confirm viability.Presence of previous large size baby, H/O DM, obesity, PCOS directs us for the oral glucose tolerance test to be done
 
 
C) The approach of expectant management can be justified in this patient thinking her to be ectopic only if she is haemodynamically stable and after confirmation with TVS and discriminatory zone with BHCG. Since expectant management of ectopic pregnancy requires medical management with injectable methotrexate, which is teratogenic; so it requires confirmation.If TVS shows adnexal mass, pseudo-gestation sac, empty uterine cavity, ring of fire on doppler, no discriminatory zone ( TVS shows no IUP at BHCG levels of 1500 IU/l) then ectopic pregnancy is confirmed and then expectant management is justified. Further mere presence of ectopic gestation does not justify expectant management.The sac size should be less than 4 cm, no cardiac activity, Serum BHCG falls more than 15% on day 1 and 4 then only expectant management is justified .
 
 
 
 
 
 
 
 
 
 
 
 
 
 
 
 
 
 
 
 
 
 
 
Posted by I N.

A) The clinical assessment should include a history and examination. In terms of the history, the duration, severity and type of the pain is important to be elicited. A cramping central abdominal pain may indicate a threatened miscarriage but a sharp/stabbing pain on either iliac fossae may be more suggestive of an ectopic pregnancy. Another symptom that would be suggestive of an ectopic pregnancy is shoulder tip pain. Furthermore, the amount and duration of bleeding should also be identified. A heavy blood loss wth blood clots would be associated more with threatened or inevitable miscarriage rather than ectopic pregnancy. In addition, an enquiry about past medical history including risk factors for an ecopic pregnancy should be made; specifically ask about history of sexually transmitted diseases, pelvic inflammatory disease and previous operations. Also past gynaecological and obstetric history is of paramount importance especially regarding menstrual cycle, previous ectopic pregnancy or history of miscarriage.

In terms of the examination, the patient's pulse, respiratory rate and blood pressure should be obtained to rule out haemodynamic instabilty. An abdominal examination is essential to assess the site and severity of pain. Moreover, signs of guarding or rebound tenderness should be looked for as these would indicate peritonitis that could be consistent of a ruptured ectopic pregnancy. A speculum examination is essntial to assess the amoun of bleeding and look at the cervical os. An open os would indicate an inevitable miscarriage and moreover, products of conception may also be seen. At the time of speculum examination, triple swabs should be obtained if the patient is at high risk of STIs. To complete the clinical assessment a bimanual examination should follow, to assess the size of the uterus (it would coincide with the weeks of gestation in a viable pregnancy), the presence of cervical excitation (that would also be another sign of peritonism) and the presence of adnexal tenderness.

B) The main factor that will influence the investigation is the patient's haemodynamic stability. If the patient is stable a full blood count and group and save sample should be obtained to assess the level of haemoglobin and group and save some blood if needed. If the patient is unstable, a full blood count and a cross match for at leats 4 units is essential. In a stable patient, a serum hCG value should be awaited to assess how far the pregnancy has progressed and also to decide whether an ultrasound scan would be valuable as with low hCG levels <1000-1500iu/L such an early pregnancy may not be visible. If there is a suspicion of ectopic pregnancy and the patient is stable a repeat hCG in 48hrs should be organised to assess if the hCG is almost doubling in case of a viable pregnancy or it is suboptimally rising that is suggestive of an ectopic pregnancy.  In a haemodynamically compromised patient where there is high suspicion of ectopic pregnancy based on the clinical assessment (e.g. shoulder tip pain, cervical excitation, adnexal tenderness) along with a positive urine pregnancy test, awaiting for an hCG value or an ultasound scan would not be appropriate.

C) Expectant management of an ectopic pregnancy can only be justified if the patient is haemodynamically stable, asymptomatic or has minimal symptoms and the hCG levels are low <= 1000iu/L. Moreover, on utrasound scan the ectopic must be small <3cm, no fetal heart beat to have been identified and there must be no or very small amount of free fluid in the pelvis.  In this particular patient, expectant management could be justified if the symptoms had settled, the hCG was not elevated and the ultrasound scan findings are consistent with what it is described above. For expectant management, the clinician needs to ensure that the patient uderstands the diagnosis and its severity and will comply with the follow up. In case of expectant management, hCG levels should be measured twice a week and it must be ensured that the levels drop by at least 50% in a week. Moreover, an ultrasound scan should be performed once a week and ensure that the size of the ectopic reduced within one week. The patient should be followed up until the hCG levels were below 20iu/L. If at any point during the follow up the patient develops any symptoms or is compromised then a surgical management should be followed.

 

answer for the essay 357 Posted by sushma S.

 

  1. I will ask her about the nature of pain and amount of bleeding. In case of pain ,whether its constant ,intermittent and colicky in nature and previous episode of such pain. Colicky nature of pain will suggest sourse is intestinal and may occur in process of  spontaneous abortion. I will ask her whether she is able to locate pain or vague , cause localization suggest sign of peritoneal irritation while vague pain suggest visceral pain .  Regarding bleeding irregular ,unpredictable and minimal in amount suggestive of ectopic pregnancy while mild or more suggestive of process of miscarriage although minimal bleeding can also happen. I will ask her any associated feature like vomiting ,fever .urinary and bowel symptom to rule out the other cause like ovarian accidents ,UTI ,PID although PID with pregnancy is rare .

     I will ask her about menstrual history LMP whether she is sure or not .regularity (irregular cycle difficult to calculate gestational age) ,amount and duration of flow. Her obstetric history should be enquired about parity , miscarriage (induced or spontaneous),mode of delivery and last child birth.  I will ask her about current pregnancy whether it was planned or unplanned , spontaneous or induced with ovulation induction.

 I will ask her contraceptive history about the method  specially about IUCD ,regular or irregular use  ,any side effect ,including emergency contraception.

 I will ask her about life style smoking ,alchohal and sexual history to know her risk of STI.  I  will ask her about previous episode of PID and any treatment taken for that. I will ask her whether she has undergone any surgical procedure or not for gynecological or surgical cause as surgery will increase the risk of adheshion and ectopic pregnancy. She should be asked about any ultrasound done in this pregnancy or not if done then review her report.

Clinical examination will include her general condition ,pulse. BP ,temperature ,respiratory rate.. per abdominal examination for any tenderness ,guarding and rigidity ,.per speculum examination for the condition of the cervix and amount of bleeding .bimanual per vaginal examination for the uterine size, adenexal mass  any tenderness or cervical movement tenderness and cervical os opened or not.

 

2 .The information available from history and examination will guide us to plan for investigation.

If she is haemodynamically stable and  as history suggest early pregnancy complication like ectopic pregnancy or miscarriage then TVS should be done to confirm .Further investigation will depend on the ultrasound finding. if its early intrauterine pregnancy with bleeding and no other associated symptom then only FBC and ABO RH factor should be done in view of expectant management.

If ultrasound finding confirm miscarriage then also no additional investigation needed other than FBC and ABO RH in few of further management either expectant ,medical and surgical management..

If ultrasound suggest ectopic pregnancy by showing no intrauterine evidence of pregnancy along with adenexal mass with free fluid in pouch of Douglas, then quantitative BHCG sample should be taken 2 sample 48 hrs apart.,along with blood group cross matching, urea and electrolytes, liver function test in case she need medical and surgical management.

If ultrasound suggest intra uterine pregnancy with possibility of ovarian cyst accident and clinical feature suggestive of vomiting then all above mentioned investigation needed in view of laproscopy and laprotomy if conservative management fails.

If ultrasound suggest intrauterine pregnancy and she has urinary symptom and bowel symptom then urine dipstix , MSU and stool for routine examination  should be sent.

 If history suggestive of  PID and she is at high risk of STI  ,and ultrasound suggest intrauterine pregnancy .then vaginal swab can be taken although pregnancy with PID is rare to occur.

 

   If  the diagnosis is uncertain and B -HCG in discriminatory zone and other surgical    cause has been rule out then diagnostic laproscopy can be done ,although it has 5% false positive and 4% false negative result.

.

 

3.Expectant management of the ectopic pregnancy is justified if the women fulfill the criteria for that and fully informed women wishes for that.

She should be managed with dedicated Early pregnancy management unit ,equipped with trained staff ,facility for the TVS, investigation,.like BHCG.

Some clinic provide 24 hrs service including weekend.

She should be haemodynamically stable ,her B- HCG should be below 1000 iu or below, if there is adenexal mass it should be below 4 cm in size and pouch of douglus has fluid less than 100 ml.

She should be agreed for the management plan ,access to follow up and ready to have operative intervention when necessary. She should be followed up with twice weekly B-HCG and weekly ultrasound TVS..  She should be followed up till her B –HCG becomes below 25 IU .

Instruction should be given when to intervene like unstable haemodynamics ,free fluid in pouch of Douglas increasing ,size of the adenexal mass is not decreasing ,B-HCG is not decreasing or pleateu .She should be provided 24 hr emergency contact number .

If she has difficulty in follow up then she can be admitted as inpatient care.

she should be supported with written information leaflets.

Essay 357 Posted by Candice W.

 

(A)

If the patient is haemodynamically unstable, I will call for help, assess her airway, breathing and circulation with a multidisciplinary team of anaesthetist, haematologist and nurses. 2 large bore cannulas are inserted and fluid resuscitation will be started immediately. Her blood pressure, pulse rate and urine output will be monitorred. Bloods for group and save, full blood count, clotting profile and beta HCG will be taken.

If she is haemodynamically stable, I will ask about the type of pain and bleeding, if POC has been passed out, any early viability scans had been done previously and if her menstrual cycles were regular. Risk factors for ectopic pregnancy will be asked such as IUCD in situ, history of PID, ectopic pregnancy, artificial reproductive techniques used or history of sterilisation.

Blood pressure, pulse rate and sign of pallor will be noted. Abdominal examination to look out for and guarding or rebound tenderness. Speculum examination to look out for amount of bleeding, passage of POC, if the os is open, any cervical abnormalities or abnormal vaginal discharge. Bimanual vaginal examination will be done to check of cervical excitation or adnexal tenderness.

Transvaginal ultrasound will be done to look for intrauterine pregnancy, adnexal masses, ectopic pregnancy, fluid in POD or molar pregnancy.

Bloods for group and cross match, Rh status, full blood count and beta HCG will be taken if not done earlier.

Differential diagnoses include : threatened miscarriage, missed miscarriage, molar pregnancy, ectopic pregnancy, pregnancy of unknown location, PID in pregnancy and other surgical causes of abdominal pain.

(B)

If the patient is pale and is actively bleeding, group and save, full blood count and clotting profile are required as she may need blood, platelet or FFP transfusion. Rh status should be checked as well as she will need anti D Ig if an evacuation is done later.

If she is haemodynamically stable, beta HCG will be taken at time of admission and 48 hours later. Decreasing trend suggests miscarriage and a suboptimal increase will suggest ectopic pregnancy.

If she is hypotensive and tachycardic, a diagnostic laparoscopy will be done to check for source of bleeding and to arrest the bleeding.

If the patient has medical history of haemophilia or clotting factor deficiencies, the value of clotting factors should be checked as well.

If there are is history of STI or PID with abnormal vaginal discharge, vaginal swab should be taken to check for infection.

Any cervical abnormalities will necessitate a cervical smear to rule out malignancy.

If there is guarding or rebound tenderness on examination, a CT should be done to look for other surgical causes of abdominal pain such as appendicitis.

(c)

Expectant management of ectopic pregnancy is justified if the patient is stable without hypotension and tachycardia, ultrasound shows minimal or no fluid in POD and the ectopic pregnancy is small with no fetal heart beat and beta HCG is less than 1000iu/l.

She must be compliant to follow up. Beta HCG twice a week and ultrasound once a week. Only if after beta HCG has reached a 50% decrease with a decrease in size of ectopic pregnancy after a week, then follow up will be weekly beta HCG and ultrasound until beta HCG is less than 20iu/l.

She must understand that medical methotraxate or surgical management may be needed if there is suboptimal decrease in beta HCG or size. There is also a risk of rupture that may need emergency surgery.

Information leaflet will be given and she has to call the helpline or return to hospital immediately if there is increase in abdominal pain or vaginal bleeding.

If all the criteria above are met, then expectant management of ectopic pregnancy in this case is justified.

 

Answer to 357 Posted by Moon M.

a)Obtaining history in regards her parity and and any previous miscarriages.Ensure that  she is sure of her date in regards LMp- and enquire about cycle regulairy.Any known medical disease or previous surgeries and allergy should be explored.Enquiring about the amount of vag bleeding and the severity of her abd pain.

I'll evaulate her general condition and take set of obsrvation Bp-pulse temp- to ensure that she is haemodynamically stable .Speculum examination to assess the amount of bleeding she is getting .high vag swab to be taken for culture and sensitivity and state of cervix if it's open if that visible and p-ossible. pelvic digital examination to assess the cervical os op-ened or closed .

Transvaginal ultrasound should be done to locate p-regnancy and confirm viability . blood should be taked for CBC,group- and save and B HCG as a base line reading.p-rogestreone can also be done as additional test if feasible. IV intravenous canulation if p-t is heavily bleeding and she is haemodynamically uunstable . According to the result of TVS in conjunction with her biochemistry  and general state of the p-atient.next step- will follow.

this p-atient is either a case of miscarrige or ?p-regnancy of unknown location.

answer to 357 Posted by Moon M.

To  continue

b)severity of symp-toms.her overall general condition if she is haemodynamically stable or not.necessity for admission or manage as on outp-atient basis .Moreover her Ulrasound finding in conjunction with her biochemistry result will affect her further management..Diagnosis are either miscarriage the typ-e of should be detrmined based on clinical finding.or  p-regnancy of unknown location or confirmed ectopic p-reg. 

C) exp-ectant management is justified if her overall general condition is good,haemodynmically stable.her symp-toms vag bleeding and abd p-ains are not comp-romising her wellbeing.er B HCG (quantitative) level are low below 1000) as base line and decreasing is subsequent reading in 48 hrs and thereafter.U/S transvaginal U/S finding is r(eassuring no cardicac p-ulsation ,small tiny preg. sac. p-t wish or op-t for conservative management ot should be involved in the decision making and her wish should be resp-ected and considered .

Posted by Pradnya K.

A healthy 23 year old woman attends the early pregnancy clinic with a 6 hour history of lower abdominal pain and vaginal bleeding. Her LMP was 8 weeks ago and she had a positive pregnancy test 2 weeks ago. Her pregnancy test is positive on admission. (a) Discuss your clinical assessment [7 marks]. (b) Discuss the factors that will influence your investigations [7 marks]. (c) She is thought to have an ectopic pregnancy and is offered expectant management. Can this approach be justified? [6 marks]

 

A )  In this woman with positive pregnancy test came with abdominal pain & vaginal bleeding, possible diagnoses of incomplete miscarriage, threatened miscarriage, ectopic pregnancy & molar pregnancy should be kept in mind.

Careful  history about severity & duration of pain should be asked. She should be asked about duration & amount of vaginal bleeding. History should be taken about passage of fleshy mass suggestive of products of conception & grape like vesicles suggestive of molar pregnancy.

Details should be asked about previous visits to EPU, case-notes, previous USS & previous serum beta-hcg reports if any.

Obstetric history should be taken including number & nature of previous pregnancies asking particularly about h/o ectopic or molar pregnancy in the past.

History should be taken about use & method of contraception ( minipill, IUCD )  prior to the current pregnancy. History of smoking & STDs should be asked.

Clinical examination: Woman’s general condition should be assessed. Temperature, pulse, blood pressure should be checked. Pallor should be looked for.

Abdomen should be inspected for any distention. Abdominal palpation should be done for any  tenderness, guarding, rigidity, large for dates uterus, collection of fluid.

Speculum examination should be done to assess the amount of bleeding, presence of POCs or grape like vesicles.

Careful & gentle per vaginal examination should be done to assess the status of cervical os, fullness of cervical canal, size & position of the uterus, presence of mass or tenderness in the lateral fornices.

b) Baseline investigations including complete blood count, U/E, coagulation studies should be done to assess & document the woman’s present condition.

Serum beta hcg levels should be checked. This gives idea about whether the levels are corresponding to the period of amenorrhoea. Serial beta hcg titres are valuable as a rise less than 50 % in 48 hours suggests a non-viable or an ectopic pregnancy. The titres may help in deciding the mode of management, for example if the levels are below the discriminatory zone ( < 1000 IU/L) & falling, an option of expectant management can be discussed with the woman.

Trans-vaginal USS should be done to check the presence of gestational sac &/or embryo, its location, cardiac activity, multiple pregnancy, molar pregnancy.

If TVS is showing viable intrauterine pregnancy with normal adnexa, patient can be managed conservatively.

 If TVS is showing a non-viable intrauterine pregnancy, decision about medical or surgical evacuation of the uterus can be undertaken after discussion with the woman.

But if there is suspicion of ectopic pregnancy on TVS , further decision of laparoscopy should be considered, provided that the patient is haemodynamically stable.

c) Certain criteria have to be fulfilled for the expectant management of ectopic pregnancy. Patient has to be haemodynamically stable & asymptomatic, serum beta hcg levels should be low ( < 1000 IU/L ) & falling , cardiac activity should not be seen in the ectopic gestation, & there should not be more than 100 ml free fluid the pouch of Douglas. Patient should have 24 hours & easy access to the hospital. Clear, written information should be given about the importance of follow up. Patient should be managed with weekly TVS & twice weekly serum beta hcg levels.

Expectant management is cost-effective than the surgical management & also the possible risks of bowel/bladder injury, anaesthetic complications are avoided. It avoids the possible side effects of the drugs used in medical management.

So in carefully selected cases, expectant management of ectopic pregnancy can be justified.

Posted by Nick M.

Clinical assessment depends upon whether or not the patient is haemodynamically stable. A  priority of initial assessment is to determine the hemodynamic status of the woman. Tachycardia, low blood pressure and reduced capillary refill time suggest shock and should be managed accordingly. If the woman was stable, I would take a careful history to assess risk factors and symptoms of ectopic pregnancy. I would confirm LMP, length and regularity of cycle. Ask about past obstetric history and enquire about risk factors for ectopic pregnancy such as previous ectopic pregnancy, history of pelvic inflammatory disease, infertility and assisted reproduction and intrauterine contraceptive devices.

A history of presenting condition should confirm amenorrhea; enquire about the character and nature of the abdominal pain, vaginal bleeding and associated symptoms such as fainting, shoulder tip-pain and diarrhoea. I would also ask if she has had any ultrasound examinations in this pregnancy.

Pulse. BP, oxygen saturations and temperature should be recorded and the abdomen examined for signs of tenderness, guarding rigidity and signs of peritonism.  Speculum examination would allow inspection of the cervix and vaginal examination may elicit adnexial tenderness, cervical excitation and even possibly palpation of adnexial mass.

B. If the woman presents with obvious signs of intra-abdominal bleeding, the only essential investigation required are a full blood count to check current Hb and a group and save to check antibody and rhesus status. 

If the woman is stable the diagnosis of ectopic pregnancy needs to be differentiated from other causes of low abdominal pain in a young pregnant woman. A urine sample should be dipped and if positive for nitrites and leucocytes, sent for culture and sensitivity. A full blood count and biochemistry sample for CRP may provide further evidence for infection.

If the cervix was dilated and products were visible on clinical examination no further investigations would be needed.

Critical to the diagnosis of ectopic and intrauterine pregnancies that are destined to fail, are serum BhCG and transvaginal ultrasound (TVS). Transvaginal US can identify intrauterine pregnancies at an earlier gestation and is therefore more specific in ruling out ectopic when compared to abdominal ultrasound.  If BhCG is below 1500iu/l (and certainly if it is below 1000iu/l)  and the TVS fails to diagnose intrauterine or ectopic pregnancy, it is important to repeat both every 48 hours until a firm diagnosis is made.  It may be that in cases of diagnostic uncertainty there is a role for serum progesterone.

C. Expectant management of ectopic pregnancies can be successful. Studies have shown success rates of about 69 percent. To be successful, the diagnosis must be made early, the woman should be haemodynamically stable, with no symptoms of pain and any adnexial mass identified on TVS should be less than 4 cm and there should be minimal amounts of free fluid in the pelvis. For the safety of the patient there should be a dedicated unit, a 24 hour contact number and the woman should be given open access. |Even if all the criteria are met, I do not think that expectant management can be justified in this case because she is clearly symptomatic.

 

Posted by QAMAR H.

 

ESSAY 357

(A) I would   obtain the  details of her  symptoms  to assess the severity ,like amount  and colour of bleeding  and  any passage of clots.Will enquire about the  intensity and type of pain.I will ask   whether she is feeling any dizziness ,shoulder tip pain or has experienced any fainting attack  .Any history of vomiting,urinay frequency or passage of vesicles will be taken.I will take her menstrual history including  LMP,regularity  of cycle,duration  and flow .I will  ask about any history of pelvic infection , tubal surgery, and use of intrauterine contraceptive device or minipills (risk factors for ectopic)  .I will  ask about her previous pregnancies  and their outcome  including  recurrent miscarriages,ectopic or molar pregnancy(risk of recurrence).Sexual history will  be taken sensitively to enquire about number of partners in previous year or  any new partner. Social history  will reveal her life style like smoking ,alcohol or drug abuse .

I will assess her general condition by measuring  her blood pressure (hypotension),pulse(tachycardia),temperature as she might need resuscitation.I  will look her conjunctiva for pallor (anaemia).I  would  perform  abdominal examination to look for any tenderness  or distension(intraperitoneal bleed).Sterile speculum examination will be  performed to confirm uterine bleeding , to exclude any dilatation of cervix and passage of product of conception . Bimanual examination will be done to assess uterine size,adnexal mass or tenderness and cervical excitation.

(B)  Most important  investigation would be transvaginal  ultrasound to confirm intrauterine pregnancy or to exclude ectopic or molar pregnancy , and its viability  and size,because further management  will depend upon diagnosis . Serum B-HCG(quantitative)will be  performed  as a base line and to look for discriminatory zone(1000-2000 iu) as pregnancy should be visualized  at this level.  If  level is low, she is haemodynamically stable and there is no intrauterine or extrauterine pregnancy   found on transvaginal scan, then serial  B-HCG  will be indicated  every 48 hours to look for doubling of level  if there is viable intrauterine  pregnancy.

Full blood count will be done  to look for anaemia as she may need surgery  and blood transfusion. Her blood group and Rh status will be performed  as she might need  blood transfusion  and anti D if she has negative blood group and her partner is positive .Urine analysis may be indicated to exclude urinary tract infection ,in case she has urinary complains . Urea and  electrolytes may be considered  if she has associated vomiting. If  she is not haemodynamically stable,ectopic is confirmed or no pregnancy is found on transvaginal scan  despite  positive pregnancy test,diagnostic  laproscopy  will be indicated  to confirm  the diagnosis  and treat simultaneously after resuscitation and necessary investigations including Full blood count,group  and  save and cross match.

(C) Expectant management  of ectopic pregnancy  involves the follow up  for 48-72 hours . If symptoms  of ectopic pregnancy occur or BHCG level rises above discriminatory zone or levels start to plateau,it should be considered for active intervention.If stable can be monitored by weekly transvaginal scan to assess reduction in size and serum BHCG  reaches to 20 iu. This management can be justified  if her pain is mild and her bleeding is slight ,ideally thereb should be no symptom,BHCG level  less than 1000 iu initially and falling,there is no blood   in poch of douglas ,less than  100 ml fluid in  pouch of douglas .Woman should be compliant ,willing for regular follow up  and is within easy access to treating  hospital. If this criteria is not fulfilled  expectant management can not be justified  and she should be managed actively.

Ans 14 B.Q Posted by Barbara Q.

From the history, constant unilateral pain rising in strength and associated with collapse would suggest ectopic whereas intermittent suprapubic pain is likely to be miscarriage. The severity of the pain and type of analgesia required will influence the urgency of treatment. Associated right upper quadrant pain or shoulder tip pain due to diaphragmatic irritation by blood is seen in ectopic pregnancy. I will enquire about the amount of bleeding as it influences the urgency of treatment. Heavy vaginal bleeding is likely to be a miscarriage as is passing products or the gestational sac per vagina.  Offensive vaginal discharge suggests septic miscarriage. Collapse with heavy vaginal bleeding is likely due to products present at the cervixcal os. Alteration in bowel habit can be due to bowel irritation by blood fro a ruptured ectopic. Urinary symptoms should be sort. Conception after assisted reproduction or the presence of IUS/IUD increases the risk of an ectopic. I will also enquire about her medical and drug history, and any drug allergies.

On examination her blood pressure and pulse should be taken to asses her haemodynamic stability. Temperature check may show pyrexia as in septic miscarriage. Abdominal examination may show guarding and rebound indicating peritonism secondary to blood from ruptured ectopic. In a skinny patient an ectopic mass may be palpable in the iliac fossa. At the gestation if the fundal height is palpated it is likely to indicate a molar pregnancy. Speculum examination will show a dilated cervix, products at the os and the amount of bleeding from the cervix.  Bimanual examination may show a mass in the adnexa or pouch of douglas. Cervical excitation although nonspecific can be seen in ectopic pregnancies. Uterine size can also be palpated and if large for dates can indicate molar pregnancies.

Suspected ectopic or miscarriage can progress rapidly to a life threatening condition as such a baseline FBC should be undertaken to note the Haemoglobin. Also group and save should be undertaken to allow crossmatch blood to be quickly available if patient becomes haemodynamically unstable. Group and save allows us to know the rhesus status of an indivdual and if interventions such as medical or surgical treatment of miscarriage or treatment pof an ectopic is undertaken the patient can receive anti D if rhesus negative. In life threatening bleed DIC or end organ damage can occur so clotting and U&E and LFDT should be taken. Scan should be done to localise the pregnancy, access viability and presence of blood in the pouch of douglas. In case of worsening pain not settling with analgesia and an inconclusive scan laparoscopy can help establish the diagnosis.

Expectant management is not invasive, require lengthy follow up and can have life threatening implications. If the patient understands the implications, is will to comply with treatment, lives close to the hospital and has a consenting adult present with her at all time who can get her to hospital if problems arise then it is a reasonable option. The patient should have minimal bleeding or pain which is not worsening. A low bHCG of 1000 and decreasing if feature of a resolving ectopic and hence allows for expectant management. Scan feature of a mass less than 3cm with absent FH and abscence of blood in the pouch of douglas can resolve spontaneously so can be managed expectantly especially if on subsequent scan the mass is decreasing in size.

Essay 357 Posted by Lutfi S.

a) As part of clinical assessment I would take a full history, examination and initial investigations. Features of the history including expansion of the presenting complaint of abdominal pain and vaginal bleeding, e.g location of pain, severity, spread exacerbating/relieving factor, vaginal bleeding, amount, clots, products, faint, which came first. Other features of the history include the patients background medical/surgical history (e.g. previous ectopics), a full obstetrics & gynaecology history, LMP and cycle length, info regarding previous pregnancies, pelvic/sexually transmitted infections, contraceptions e.g. coils, together with a social history.

Examination would include assessing the patients ABC status. If airway and breathing are stable her pulse, BP, Capillary refill and oxygen saturations should be assessed. I would then proceed to examine her abdomen, assessing the pain, severity, guarding, rebound and location. A pelvic examination would then be performed, examining the external genetalia assessing the amount of bleeding. A speculum examination would be performed followed by a gentle bimanual examination. The cervix should be assessed to see if open and amount of bleeding coming through, together with examination to check for cervical excitation/tenderness. A gentle examination of the size, position and mobility of uterus is performed together with any signs of adenxae tenderness/masses.

b) As part of the examination I would want to assess her BMI and urine dipstick.

Other initial investigations that would be performed while examination would be swabs to check for pelvic infections including a high vaginal swab and chlymydia swab and this would be performed with the speculum examination.

Factors which would influence investigations would depend on how stable the patient is. If the patient is haemodynamically unstable priority is immediate resuscitation and treatment (theatre if unstable). Provided the patient is haemodynamically stable I would perform blood investigations including a FBC (to check haemoglobin and platelet) U&E, LFT's (if ectopic and considering medical management), a G&S to check blood group if blood needed or for rhesus status. A quantative serum BHCG would be taken if suspected ectopic and to check if level is in discriminatory zone.

Imaging would include an ultrasound (transabdominal and transvaginal) of the pelvis to check location, gestation and viability of pregnancy. If serum BHCG is above discriminitory zone and pregnancy not seen in uterus we can proceed to a diagnostic laparoscopy to assess pelvic/abdominal cavity for location of pregnancy and proceed to treat ectopic if necessary. Serial BHCG can be taken if levels below discriminitory zone  and patient is stable.

c) Expectant management of ectopic pregnancy can be justified if the patient is asymptomatic and haemodynamically stable. The patient should have an ultrasound which confirms pregnancy is ectopic. It is assumed the ectopic pregnancy is a failing pregnancy which will resolve spontaneously. She should have open access to a gynaecology ward with contact details available. The BHCG level should be tracked every 72 hours with an expected fall. It is recommended the starting BHCG level should be <1000 IU. The BHCG should be followed up until the level falls <20IU.  There should be minimal amount of blood in the pouch of Douglas on ultrasound scan. Should the BHCG not fall or if patient is symptomatic she should be informed she may need alternative treatment including surgical management. She should have a responsible adult with her and they should be informed of the possible diagnosis and to bring patient in if deviation with any clinical signs or symptoms from ectopic pregnancy. If pregnancy resolves information should be given for early USS in next pregnancy to confirm location of pregnancy and risk of ectopic rises after previous ectopics.

essay 357 by NA Posted by naila A.

a) Details of symptoms should be explored. This should focus on whether the onset of pain was before the start of bleeding or after the start of bleeding. If the pain started after the bleeding it is more likely due to ectopic and if pain started before the bleeding it is more likely miscarriage. The amount of blood loss is also important if it is heavy with blood clots it is suggestive of a miscarriage. Minimal port-wine coloured discharge would suggest ectopic pregnancy. Obstetric history is important to know the number of children and their mode of deliveries. Her desire of retaining fertility should also be ascertained. History of tubal damage and infertility should be determined. History of ovulation induction should also be obtained as it is also a risk factor for ectopic pregnancy.

         Abdomen should be palpated to see if there is generalized tenderness or localized pain in one of the iliac fossa. With ectopic pregnancy the pain is more likely to localize in one fossa. Vaginal examination in case of ectopic pregnancy reveals closed cervix and tenderness in one iliac fossa. In case of threatened miscarriage the cervix is closed and in case of inevitable or incomplete miscarriage the cervix is open.

        b) A full blood count should be done to check haemoglobin to ascertain if there is significant haemorrhage, and if the Hb is <9/g/dl she may need blood transfusion. The blood group should be determined and serum should be saved so that in case if she needs blood transfusion it can be transfused. Rhesus status should be determined so that if anti D is required it can be provided. The beta  human chorionic gonadotrophin(HCG) concentration should be determined to aid in the diagnosis of ectopic pregnancy. Transvaginal ultrasound should be done as she has positive pregnancy test two weeks ago so an intrauterine or ectopic pregnancy is likely to be seen. She is advised to stay nil by mouth and I/V access should be obtained. Level of beta HCG and report of TVS will determine further management. If beta HCG is more than 1000 and pregnancy is not seen serum progesterone should be done to assess the  risk for ectopic pregnancy. If it is >60, she is at high risk of ectopic pregnancy she should have a repeat scan as soon as possible or diagnostic laparoscopy should be planned. If beta HCG is <1000 and progesterone is >60, the risk of ectopic is low and an ultrasound should be repeated when HCG is more than1000. If HCG is less than 25 and progesterone is less than 25 she should be reassured and blood /urine should be repeated after one week. If progesterone is between 25 to 60 and HCG is >25 there is high risk of ectopic and she should have repeat bloods in 2 days.  

          c) Most ectopic pregnancies resolve spontaneously. Now their early diagnosis has resulted in over-treatment. Data from the management of pregnancies of unknown locations which were followed further shows that most of the abnormal pregnancies were ectopic, therefore if a pregnancy of unknown location is considered more likely to be ectopic ,it is justified to manage her expectantly. In case of known ectopic there is little justification of this management comparing its risks with high efficacy of methotrexate and surgical treatment. RCOG states it acceptable only if it involves little minimal risk to the women therefore if it is considered only in those cases which are selected very carefully, where woman is asymptomatic and has rapidly falling HCG levels

ans to saq 357 Posted by munazza R.

a)-history obtained regarding severity of symptoms,type of pain,colour and amount of bleeding,passage of clots with intermittent cramping.

any associated vaginal discharge,bladder and bowl symptoms,fever ,rigors,nausea ,vomiting,shoulder tip pain,breathlessness and fainting .

any risk factors for suspected ectopic pregnancy.

general physical examination including pallor,dyspnoea,B.P,puse,temperature.

abdominal examination to look for site of pain,unilateral or generalised,tenderness ,guarding,any palpable mass.

vaginal speculum examination to look for atate of cervix,dilatation of oss,any clots in vagina,any vaginal discharge.swabs taken for infection screen.

bimanual examination to look for any adnexal mass or tenderness.,

investigations include FBC,SERUM BHCG,C-REACTIVE PROTEINS,DIFFERENTIAL LEUCOCYTE COUNT,SERUM PROGESTERONE,MSU for C/s and microscopy,blood group and save.Trans vaginal ultra sound as per hospital protocol.

b)-diagnostic and therapeutic algorithm of hospital.

quality of ultrasound equipment

experience of staff in performing ultrasound,laparoscopy and laparotomy.

severity of symptoms and haemodynamic stability of patient.

presence of physical factors e.g uterine fibroids,multiple pregnancy

uncertainity of diagnosis which may lead to proceed to laparoscopy or laparotomy

past surgical history,medical history,and wishes for future fertility.

c)-it is justified in selected cases,after obtaining infromed consent,compliant patient,living within easy access to the hospital.

It must involve minimal risk to the patient,she should be clinically stable,with an ultrasound diagnosis of ectopic pregnancy,,less than 100 ml fluid in pouch of douglas and no evidence of blood in it,ideally size of ectopic should be less than 4 cm and no cardiac activity and HCG levels less than 1000 iu/L and rapidly falling.

patient should be able to come for follow up twicw weekly initially,and then weekly thereafter for BHCG and TVS till BHCG is less than 20 iu/L.

patient should understand and agree with the need for further intervention if she becomes symptomatic or expectant management fails

she should be provided with 24 hours telephone contact lines,addresses of support groups,websites and information leaflets

357 Posted by Sailaja C.

 

 

(a) Discuss your clinical assessment [7 marks].

 

 

The clinical symptoms are suggestive of miscarriage (threatened or inevitable) or ectopic pregnancy. Molar pregnancy is another possibility.

 

Assessment is made to identify overt bleeding, and by recording pulse and blood pressure. If the bleeding is severe and woman's haemodynamic status is compromised, resuscitative measures are commenced by obtaining venous access.

 

If her haemodynamic status is stable history is taken regarding nature of pain, location, radiation. Nature of the bleeding is noted if she has passed clots , parts of conception or grape like vesicles.

 

Menstrual history is taken which includes, regularity, cycle length, flow, dymenorrhoea and LMP.

Details of conceptions are taken if she conceived after assisted reproductive methods as assissted conception is a risk factor for ectopic pregnancy.

Obstetric history is taken about the details of her previous pregnancies and mode of deliveries.

History is taken regarding previous ectopic pregnancy or previous treatment for PID which are risk factors for ectopic pregnancy. Contraceptive history is taken as use of Mirena coil and minipill predispose to ectopic pregnancy.

Pulse and BP are recorded. Assessment is done to identify pallor or signs of hypovolaemia.

Abdominal examination is performed to identify any tenderness, guarding and rebound tenderness.

Speculum examination is done to assess the amount of bleeding per vagina, and for the presence of products of conception in the cervical canal suggestive of inevitable miscarriage.

Bimanual vaginal examination is done to assess uterine size and any adnexal mass.

 

B)

Investigations are directed according to findings at clinical assessment and ultrasound findings.

 

Severity of the bleeding is a factor for doing primary invesitigations like FBC, grouping and saving are required as the woman is bleeding. Crossmatching may be required if she presents with hypovolaemic shock.

 

History and her clinical presentation is another factor which decides the requirement of transvaginal ultrasound scanning to identify the location of pregnany intrauterine or extrauterine and to assess fetal viability.

 

Rh typing is done to decide if she needs administration of Anti D Ig if she requires evacuation or if ectopic is diagnosed.

 

The other factors which direct the investigations are findings at transvaginal utrasound scan. Serial serum B- HCG levels are required if ectopic pregnancy is diagnosed with minimal symptoms to facilitate expectant management.

 

If no GS is seen in the ultrasound scanning, further scan is required after one week and serum B HCG levels after 48 hours as less than 66% rise in B HCG levels indicates failing pregnancy.

 

 

c)

If the woman is asymptomatic and haemodynamically stable expectant management is employed.

 

The serum B-HCG levels should be low and falling (<1000iu/l)

 

Expectant management can be undertaken if TVS shows < 100ml blood in pouch of Douglas

 

There should not be fetal heart pulsations in ectopic mass during trans vaginal scan.

 

Low and rapidly falling HCG levels indicate high likelihood of successful expectant management

 

 

Understanding the importance of compliance with follow-up and easy access to the hospital are the criteria for expectant management.

 

Rupture may still occur under these circumstances. 

Posted by sadaf A.

a)clinical assessment with include nature of abdominal pain,amount of bleeding,passage of clots or any tissue.

confirmation of LMP,regularity of cycle,h/o previous pregnancies ,their outcome.if there is h/o previous ectopic ,what treatment was given for that,which sided ectopic pregnancy was that?

h/o STIs and contarception is imporatant as with previous h/o PID and IUD in situ chances of ectopic pregnancy are increased.h/o previous surgeries including laprotomy,laproscopy,salpingotomy or salpingectomy.

on general physicial examination,will check for general contion,BP,pulse and temperature.on abdominal palpation will check for guarding,rigidity and tenderness in which quadrant of abdoman.

on bimannual vaginal examination,uterine size assessment,cervical os dilitation,cervical excitation,adenaxial fullness will be assessed for ant mass.

b)fbc should be done to assess hb%,blood group and Rh status should be sent,in case of Rh negative kleuheir test may be sent.LFT and RFT sholuld be done as base line in case pt need surgery.

beta hcg should be done to have base line reading for expectant management.if bhcg is doubling in 48hours it is referring to healthy pregnancy.

progesterone levle can be sent as >60mmol/l shows healthy pregnancy.and <20mmol/l shows failing pregnancy.

ultrasound pelvis should be done to rule out intra/extra uterine pregnancy,vibility and any sign of free fluid if suspecting ruptured ectopic.

as above pt is in sever pain for last 6 hours and u/s is inconclusive,diagnostic laproscopy should be done

c)as pt is suspected to have ectopic pregnancy and expectant management is offered to her,to fit in criteria of expectant management her beta hcg should be <1000iu/ml.she should be vitally stable.no sign of rupture ectopic.on ultrasound there should not be any fetal cardic activity and if free fluid is in pelvis it should not be more that 100ml in pouch of douglus.

pt should be informed about risk and benifits of expectant management,risk of rupture and need for emergency surgery as well.she will have repeated tests for hcg atleast twice weely till 20iu/ml and then once weely till it become less that 5mmol/ml.

u/s pelvis for ectopic pregnancy size should be done once weekly to assess reduction in size.

at last i suggest if she is fullfilling criteria for expectant management for hcg level,u/s findings and general condition along with her wishes of compliance with follow up,it is justified to manage her expectantly.

Nee Posted by nee P.

A)I will seeher general condition , stability & measure her pulse,Bp ,resp rate,temperature & consciousness ,as its important to stabilize her if unstable, before proceeding  further.Once she found to be stable,I will take her short history regarding the bleeding like onset , duration & aggravating factors. She should be asked about the amount of bleeding, any clots or products found along with bleeding as she might have spontaneous miscarriage. Regarding pain I will ask if it is intermittent or constant as it may suggest if uterine contractions or other associated pain. I will examine her abdomen & I will inspect for any distension , palpation for rigidity & rebound tenderness which is a feature of haemoperitoneum. Site of pain can be assessed if no obvious distension found, as unilateral pain with or without PV bleeding may an impending sign of ruptured (tubal) ectopic pregnancy. Per speculum examination should be done to see any bleeding through os, amount of bleeding. Any products or clots found should be reviewed for products of conception & kept for Histopathological reporting. Per speculum examination help to find out open or closed os. Bimanual examination will help to assess the size of the uterus & any tenderness or mass in fornix will give a picture of tubal pregnancy or ruptured cyst or abcess.

B) Severity of the bleeding & her general status as, if she is unstable investigations should be in the line of life saving measures like FBC, Haematocite, oxygen saturation, group & Rh Typing. Otherwise FBC, Group & Rh typing, USG pelvis as well as HIV, HbsAg as routine tests. USG finding again an important factor , if she is found to be having ectopic pregnancy , she will require investigations like serial B HCG titre & serial USG depemding upon the mode of management . This also will suggest viability of the gestation. Similarly serum progesterone may help for viability of pregnancy. Past obstetric history like recurrent pregnancy loss  , Antiphospholipid syndrome can be suspected & blood can be sent for antiphospholipid antibodies. Products of conception can be sent for karyotyping if it is recurrent pregnancy loss. Similarly laparoscopy may be used in selective cases if ectopic pregnancy found on USG.

C) Ectopic pregnancy , if tubal pregnancy can be followed up as many tubal pregnancies resolve spontaneously.But pregnancy like cervical or abdominal, expectant management is not advised as risk of rupture & bleeding is high. So they need immediate care in the form of medical or surgical management. If her B HCG titre showing downtoward trend below 1000mIU/ml in tubal ectopic pregnancy can be advised to follow expectant management. But she should have adequate information , counseling & knowledge about signs of rupture should be provided.as there are instances where B HCG titre less the 20mIU /ml has lead to rupture of tubal pregnancy. So she should be under regular monitoring. If B HCG titre is more than 1000miu/ml & increasing trend , her pregnancy should be managed medically or surgically & expectant management is not advised as risk of rupture is high.

Ectopic Posted by Samira  K.

a-Clinical assessment includes history and examination.Detailed history of pain and vaginal bleeding should be taken and risk factors of ectopic should be explored.

In history Type and severity of pain should be explored as mild period like pain or severe or dull continous pain .This information will help us in determining if this is funtional pain or related to pathology especially ectopic.radiation of pain as ectopic pain sometimes radiates to legs .Associations of pain with gastrointestinal symptoms like abdominal bloating,nausea,vomiting and diarrhea might point towars gastroenteritis but these symptoms might be present in ectopic also . Dizziness and history of fainting point towards ectopic.If she has taken any analgesia for this pain and effect of analgesia as pain of ectopic will not be relived by simple anagesics.Any history of frequancy,urgeny or burning micturation point towards urinary tract infection.Severe intermittent pain associated with nausea and vomiting might be due to ovarian torsion.

History of amount of bleeding and its associations as it might be post coital bleeding.Any history of blood clots and passage of tissue point towards miscarriage.

Risk factors for ectopic  like previous ectopic,previous history of PID or STIs and assisted reproduction should be excluded.

Eamination starts from genaral look of women if she is pale,dizzy might be due to anemia or anemia due to ruptured ectopic.Pulse,B.P,Respiratory rate should be checked if hypotensive and tachycardic means she is in hypovolemic shock necessitating emergency management.Abominalexamination to check tenderness,fullnes or any mass as peritoneal irritation due to ruptered ectopic or ovarian tumors might be discovered Speculum examination to see any cervical lesion which might be the cause of bleeding .Bimaual examination to feel cervical motion tenderness.adnexal tenderness or fullnes .

b-Factors in history and examination will influence our investigations.It can be divided into two categories depending on condition of patient.If she is hemodynamically stabe with normal B.P ,pulse and resiratory rate and not fainting,then 1st line of investigation will be transvaginal Scan and repeat pregancy test .Findings of Scan will direct us what is needed next.If Scan report reveals normally looking intrauterine gestational sac and no evidence of ectopic or ovarian cyst then MSU will be send to rule out UTI or if GIT symptoms stool microscopy can be requested .

If Scan reveals Ovarian cyst of >5 cm and kind of pain is severe intermittent sharp with nausea and vomiting then torsion can be suspected and acted on accordingly.If Scan reveals intrauterine products of conception then miscarrige is suspected and managed accordingly.If empty Uterus is identified and there is evidence of extrauterine pregnancy then BHCG quantitative should be requested as a baseline for management and follow up.If empty uterus and no extrauterine pregnancy identified thenBHCG quantitative for discriminatory zone as if more than 100 units according to unit protocol and no intrauterine sac identified then most likely it is ectopic . serum progestrone can be reguested if it is less than 20ng/ml likely to be nonviable pregnancy and if it is more than 60 strongly related to viable pregnancy.On Scan we might find out nonviable intrauterine pregnancy and managed accordingly .If the patient passed any tissue it should be send for histopathology for confirmation of intaruterine prgnancy and to rule out molar pregnancy.

If the patient is fainting and hemodynamically unstale then We can perform abdominal scan quickly to confirm hemoperitoneum and send FBC and group and save together with BHCG Quantitative as baseline for postoperative follow up if needed and to see extent of anemia for requirement of blood transfusion.

c-Expectant management of ectopic is justified if patient is clinically stable and asymptomatic with ultrasound showing adnexal mass les than 4 cm . Less than 100 ml of free fuid and no evidence of blood in pouch of douglas.BHCG less than 1000 iu/l initially or falling rapidly by monitoring of hcg twice Wkly and wkly transvaginal scan .Ideally hcg should fall 50% of its initial level wthin 7 days.and reduction in size of ectopic.Then wkly scans and HCG until it is less then 20iu/l.

Expectant management can be offered to a women after counselling and making sure that she will be compliant with follow up,having easy access to hospital or early pregnancy unit any time in 24 hrs.Partner is avaible and both of them understand the risk of rupture during expectant management and informed consent taken.

 

Posted by sindhu K.

 

a.  First I will assess whether the patient is stable or unstable by checking mental status of the patient [ anxious, irritable, apprehensive, response to call], pallor, pulse[rate , volume],blood pressure, respiratory rate, oxygen saturation because  patients with ruptured ectopic, inevitable abortion, incomplete abortion can be presented in unstable condition due to excessive blood loss and in that case  need to be immediately resuscitated and surgically managed.

 

                       If the patient is stable I will proceed to a detailed history and examination. From the LMP given and  asking for the regularity of cycle gestational age can be assessed.then I would ask the characteristic of the pain like severity[ severe pain associated with ruptured ectopic], unilateral or bilateral or midline[ectopic-unilateral, abortion –midline], radiation of pain like shoulder tip pain associated with ruptured ectopic. Then I would ask for the nature of bleeding. Heavy bleeding usually associated with inevitable or incomplete abortion. Brownish blood associated with missed abortion. History of passage of clots-heavy bleeding associated with clots. History of passage of fleshy material –incomplete and complete abortion. Rarely decidual cast passed in ectopic pregnancy. Constitutional symptoms nausea and vomiting in favour of ectopic pregnancy. Also history of giddiness and syncope.

              Now I would ask for risk factors of ectopic pregnancy like ovulation induction, assisted conception, previus ectopic pregnancy and if the type of management, abdomimnal and pelvic surgeries, PID, tubal surgeries, contraceptive use especially IUCD and minipill as it will give a clue to the diagnosis of ectopic pregnancy.

             Now I will ask for symptoms pertaining to urinary and gastrointestinal system to rule out other cause of pain.

 

              Now I would proceed to systemic examination as I have already finishaed the general assessment of the patient  in the beginning. I will assess the cardiovascular and respiratory system to detect any co existing pathology and this is especially important because patient may need to be taken now or later to the theatre.during abdominal examination I would look for tenderness and guarding- localized or generalized or board like rigidity which is a feature of ruptured ectopic,  uterus palpable or not if palpable size of uterus –size is more in vesicular mole., any other mass palpable like an ovaian cyst causing pain. I will do a speculum examination and assess the bleeding[ colour , active bleeding present or not  if present the amount of bleeding because if severe bleeding surgical  intervention needed immediately, cervical os open or closed- open means inevitable or incomplete abortion, products protruding, local lesion in the cervix causing bleeding. Now I will do vaginalexamination and assess forniceal fullness,tenderness, mass, cervical exitation sign[ features of ectopic]

 

b.   if the patient is unstable as part of assessment and preparing for surgical intervention  I  would do FBC, blood group and rh, cross match 4 pints packed RBCS, liver function tests, renal function tests, coagulation profile[ to diagnose DIC], urine routine,HIV,HBSAG, VDRL screening.

                           If the patient is stable, FBC, blood group and rh [if not previously done ], urine routine to look for any infection will be sent and an ultrasound abdomen and pelvis to look for gestational age, location of pregnancy, viability of pregnancy, free fluid in the pelvis, adnexal mass . if no pregnancy located, I would ask for serum beeta HCG . with a serum beeta hcg of 1500iu/l and above we  should be able to see gestational sac for intrauterine pregnany. Exeption include multiple pregnancy. If beeta HCG is below 1500iu/l[ discriminatory zone], it should be repeated in 48 hours.less than 66% increase indicates nonviable pregnancy either intrauterine or extrauterine. If beeta hcg is above discrimimnatory zone and no intrauterine pregnancy seen on ultrasound I would do a diagnostic laparoscopy.

 

 

c.  expectant management is feasible in case of ectopic pregnancy if the following criteria are fulfilled

  1. patient stable and asymptomatic
  2. beete HCG less than 1000iu/l and falling
  3. patient readily understand that it needs regular follow up, ie, twice weekly beeta hcg in the first week and then weekly and weekly USG. Beta hcg should fall more than 50% in the first week and size of gestation sac should reduce.
  4. patient readily understands chance of failure and risk of rupture of ectopic pregnancy and ready to report to casualty any time and also for surgical intervention.
  5. hospital have the facility for 24 hours emergency service.
  6. size of ectopic mass less than 3 cm .
  7. free fluid is less than

the  patient in question is symptomatic with pain and bleeding . so expectant management is not justified.

reply Posted by S M.
  1. A detailed history enquiring about nature and site of pain be taken. Any lower abdominal pain especially in iliac fossa might point towards ectopic pregnancy. Generalized and upper abdominal pain might point towards intraperitoneal bleed making ectopic highly likely. Pain preceded by bleeding may be more suggestive of ectopic pregnancy. Ask about amount and colour of bleeding. Fresh heavy bleeding with clots might be suggestive of miscarriage more than ectopic. Pattern and regularity of menstrual cycle will help in estimating gestation accurately. Previous obstetric history be taken. Previous ectopic pregnancy will make suspicion of ectopic highly likely. Enquire about contraceptive history, previous PID, STD or use of IUCD. BP and pulse be checked. Abdominal examination is done to check for any masses, tenderness, guarding and rigidity. A rigid abdomen will make intraabdominal bleed highly likely. Speculum examination be done to assess the amount of bleeding and to remove any products. Vaginal examination done to assess for adnexal mass, tenderness and cervical motion tenderness as all of these features will point towards ectopic as likely diagnosis.
  2. Initial investigations will depend on the heamodynamic stability of patient. Full blood count and Group and save is done if patient is having heavy bleeding but otherwise stable. Blood should be cross matched if patient is haemodynamically unstable.  Vaginal Ultrasound should be requested to rule out ectopic pregnancy. If intrauterine gestation sac with yolk sac seen on scan, ectopic will be unlikely. If scan is inconclusive, serial Bhcg be done.  Levels rising by more than 66% in 48 hours will point more towards ongoing intrauterine pregnancy and scan can be repeated in a week time. However it also depends on the symptoms of patients. In case of clinical picture highly suggestive of ectopic pregnancy, diagnostic laparoscopy can be performed with the intention of salpingectomy/salpingotomy if ectopic is confirmed.
  3. Expectant management can be justified if patient is asymptomatic, is haemodynamically stable. There is no evidence of live ectopic on scan and no free fluid in pouch of Douglas. The other important factor would be Bhcg levels. Levels more than 3000 will require active intervention as risk of rupture is significantly increased. She will need regular follow ups with serial monitoring of Bhcg followed by weekly scans until diagnosis is confirmed or she becomes synptomatic. She should have ready access to hospital emergency department. Contact numbers should be given and written and verbal advice given on when and how to contact.  
dr.lalitha devi Posted by lalitha N.

Ectopic pregnancy is to be considered the foremost in such presentations as this causes significant maternal mortality and morbidity.

The nature of pain, severity of pain and location of pain need to be enquired.

Severe bleeding and passage of tissue would favour a diagnosis of miscarriage

Any fainting attacks, h/o shoulder tip pain would indicate intra abdominal bleeding.

Obstetric history :

parity ,  ectopic pregnancies in the past indicate increased risk for recurrent ectopic.

Past h/o  sexually transmitted diease, and treatment taken would point towards pelvic inflammatory disease and increased risk of ectopic pregnancy.

Present contraceptive history of using IUCD or progesterone only pill increase the risk of ectopic pregnancy

Surgical history : if patient has undergone any pelvic surgery in the past, especially tubal surgery, as subsequent adhesions may favour an ectopic.

Examination:  

general examination for pallor, pulse , blood pressure measurements  and respiratory rate to assess hemodynamic stability.

Abdominal examinationto look for tenederness, guarding and rebound tenderness,  if present, indicate peritonitis and be consistent with a diagnosis of ruptured ectopic.

Speculum examinationto see the status of cervix. If os is open with heavy bleeding or if products of conception are seen in the cervix it would be be an inevitable miscarriage.

A bimanual examination to assess  the size of the uterus, uterine tenderness, adnexal mass and tenderness.

Cervical motion tenderness  would indicate peritonism and favour diagnosis of ectopic pregnancy.

 

 

Investigations

-are guided by clinical findings  and haemodynamic stability

Clinical findings may necessitate resuscitation and surgery without further investigation.

Firm diagnosis of miscarriage may be reached without further investigation.

 If the woman is stable and diagnosis is uncertain:

FBC, group and  save

Hemoglobin concentration and blood group aid  the management.

Repeat FBC with falling Hb levels indicate internal bleeding and will decide the urgency of surgical intervention.

RH –ve group women need anti D immunoglobulin

Renal function tests and liver function tests if condition deteriotes.

Trans-vaginal ultrasound

scan will confirm intra-uterine pregnancy and fetal viability.

May detect ectopic pregnancy or free intra-peritoneal fluid.
If there is no intra-uterine pregnancy, possible diagnoses are very early gestation, complete miscarriage or ectopic pregnancy.

 

Criteria for expectant management are

Patient needs to be asymptomatic and clinically stable .

HCG levels need to be low and falling  --- (< 1000iu/l)

No evidence of or < 100 ml blood in pouch of Douglas and no fetal heart in the ectopic mass.

Women selected for expectant management need to know the importance of compliance with follow –up . should have easy access to hospital in case of emergency.

Must be followed up with serial HCG levels and weekly transvaginal scans to ensure falling HCG levels   ideally less than 50% of its initial level within seven days and decreasing size of the ectopic mass. There after follow up on basis of weekly HCG levels and transvaginal scans until serum HCG levels are less than 20iu/l as tubal rupture can occur even at low levels of HCG

 

In this case as patient has bleeding with abdominal pain she may not be suitable for this modality of management as bleeding may increase and  patient may become unstable hemodynamically.

the demands of long follow up of expectant management is not justified as the situation may worsen anytime and an immediate surgical intervention would be needed,

 

Correction Posted by Samira  K.

Dear Dr Paul

Please check my essay as essays sent after me are being checked.I am waiting for the correction

Thanks

Posted by Heba R.
Dr, Paul ,please could you tell me where is the model answer for this question? Thanx.
Posted by Heba R.
Dr, Paul ,please could you tell me where is the model answer for this question? Thanx.
Essay not corrected Posted by S M.

Hi Paul

My Essay did not get corrected. I posted it on saturday. I can get idea from the marking scheme but would have nice to get essay marked so I can work on my deficiencies. Many thanks

Posted by sadaf A.
Dear paul Plz mark my essay Regards
SM Posted by S M.

I really can't see my answer marked!!